LSAT and Law School Admissions Forum

Get expert LSAT preparation and law school admissions advice from PowerScore Test Preparation.

 Administrator
PowerScore Staff
  • PowerScore Staff
  • Posts: 8917
  • Joined: Feb 02, 2011
|
#23985
Complete Question Explanation

Weaken. The correct answer choice is (B)

Answer choice (A): Even if the government does not provide a full 100 percent of the program’s funding, it does not change the fact that government funding is needed for the program to continue. This answer thus does not weaken the report authors’ position.

Answer choice (B): This is the correct answer choice. It suggests that the report authors’ quick-fix approach will not work since the problems will continue precisely because they lack a coherent vision. As a result, there will be no end to the problems because the report authors are not getting to their roots. Moreover, if serious problems continue to surface, the report authors’ attempt to create a reputation for competence would also fail, and the government funding needed for the program would in turn be lost.

Answer choice (C): If it is impossible to sustain a coherent vision, then surely there is nothing wrong with the position taken by the report’s authors.

Answer choice (D): This is not the correct answer since it does not state why the government has threatened to cut off funding for the program. As such, it does not undermine the report authors’ position.

Answer choice (E): If the program has a worse reputation for incompetence than it deserves, then it seems like the report authors’ position is correct, in that they should be working to regain this reputation first. In any case, it does not weaken the authors’ position.
 chian9010
  • Posts: 81
  • Joined: Jun 08, 2018
|
#47340
Dear Powerscore,

I wasn't sure if I understand your explanation correctly. From the stimulus, I thought the report's author agrees with critics point of view (line 6: "in response the report's authors granted that the critics had raised a valid point") but he explains that the priority now is to get government funding first. He didn't say he won't accept the critics suggestions to produce a coherent vision for the future of the program but in order to produce a coherent vision, he needs to get the funding first.

priority steps:
1. regain reputation for competenece -> 2. get government funding -> 3. do anything to solve the problem

I chose answer choice A because if the government doesn;t provide 100 percent of the funding, then they might skip the first two steps and ask funding from other sources and then try to solve the problem.

I didn't chose answer choice B because the report's author already agree ("granted") with the critics' point of view. He just needs to prioritize the tasks.
 James Finch
PowerScore Staff
  • PowerScore Staff
  • Posts: 943
  • Joined: Sep 06, 2017
|
#47368
Hi Chian,

The issue with answer choice (A) is that the amount of the funding isn't specified, and could be anywhere from 1-99%. If the government is providing 99% of the funding, then the report's author still has a pretty valid argument, and no argument at all if it's 1%. This uncertainty means that (A) doesn't necessarily work to strengthen the counter-argument.

(B) works by making a coherent vision for the program a necessary condition for solving its serious problems, meaning that without a coherent vision the program would never regain a reputation for competence and thus be cutoff from the government funding necessary to the continuance of the program. The conditional diagram would look like:

Report's author's argument:

Program Continuing (PC) :arrow: Continued Government Funding (CGF)
CGF :arrow: Regain Reputation for Competence (RRC)
RRC :arrow: Solve Significant Problems (SSP)

Thus:

PC :arrow: CGF :arrow: RRC :arrow: SSP

and the contrapositive:

SSP :arrow: RRC :arrow: GF :arrow: PC

B gives us:

SSP :arrow: CV, thus:

CV :arrow: SSP :arrow: RRC :arrow: GF :arrow: PC

This connects the criticism of the report (lack of a coherent vision) to the ability of the program to survive.

Hope this clears things up!
User avatar
 LSAT2HARD
  • Posts: 14
  • Joined: Jan 03, 2021
|
#82945
I am very confused between B and D. I listed B and D as both contenders, and then I imagined a scenario where I will apply these answers.

For Answer B:
Report's author conclude: the program needed continued government funding, and that to get such funding the program first needed to regain a reputation for competence.
Critics: The program will continue to have numerous problems because it lacks a coherent vision for its future.
Report's author: I agree, as I said that you have "raised a valid point" but to do anything (establish a coherent vision or whatever), the government still need to continue fund so that we can do it
Thus, under this scenario, I don't think answer B can attack the report's author

For Answer D:
Report's author conclude: the program needed continued government funding, and that to get such funding the program first needed to regain a reputation for competence.
Critics: You need the funds and the government has not cut off funding yet even though they threatened to do so (so now do your job; focus on coherent vision for the future and regain your reputation)

Thanks in advance.
User avatar
 KelseyWoods
PowerScore Staff
  • PowerScore Staff
  • Posts: 1079
  • Joined: Jun 26, 2013
|
#83012
Hi LSAT2HARD!

Be careful with how you are interpreting the argument of the report authors. Their specific argument is that they didn't produce a coherent vision for the future because regaining a reputation for competence is necessary to receive government funding. So basically they are saying that they were justified in not provided a coherent vision for the future because they are focused on obtaining government funding.

Answer choice (B) attacks this argument by saying that having a coherent vision for the future is necessary for regaining a reputation for competence. If having a coherent vision for the future is necessary for regaining a reputation for competence, and regaining a reputation for competence is necessary to receive government funding, then that means that having a coherent vision for the future is necessary for receiving government funding as well. (Check out James's post above for the conditional diagramming of this logic.) This would undermine the report authors' argument that they are justified in not having a coherent vision for the future because they need government funding. If an action is necessary for your goal, then you cannot argue that you did not complete that action because you were trying to achieve that goal.

Answer choice (D) just says that the government has threatened to cut off funding. But this is not relevant to the argument because the argument is not about whether they will receive government funding. Instead, it is about whether they are justified in not providing a coherent vision for the future because they need government funding.

With Weaken questions, it is always important to focus on the precise conclusion of the argument that you are trying to undermine!

Hope this helps!

Best,
Kelsey
User avatar
 sdb606
  • Posts: 78
  • Joined: Feb 22, 2021
|
#85792
I'm still trying to work through why C is wrong.

The way I personalized C, I imagined the critics saying to the authors, "Back then, you had a coherent vision and you were competent, as evidenced by your use of the word 'regain.' Therefore, to get competent again, you need a coherent vision like old times. But you need a different coherent vision than the old one that was impossible to sustain."

C doesn't say ALL coherent visions are impossible to sustain. Just "that vision."
 Adam Tyson
PowerScore Staff
  • PowerScore Staff
  • Posts: 5153
  • Joined: Apr 14, 2011
|
#86524
The problem isn't with what answer C says, sdb606, but with what it does not say. It fails to respond at all to the position of the authors, which is that focusing on pragmatic solutions will allow the program to regain a reputation for competence. Their argument is "we need funding, and for that we need to appear competent, so we had to focus on solving those practical problems rather than on the broader vision." Answer C never addresses that argument by saying either that focusing on the practical problems is unnecessary to regain that reputation, or that doing so will be insufficient for that outcome. Since the question asks us to undermine the position of the authors, and this answer essentially ignores that position, it does nothing to answer that question.
 leslie7
  • Posts: 73
  • Joined: Oct 06, 2020
|
#90054
Kelsey, out of all of these I liked your response the best - I usually find yours the most clear and easiest to follow ! = )

But your explanation says that AC, B " attacks this argument by saying that having a coherent vision for the future is necessary for regaining a reputation for competence"

I don't see this in AC, B.

It says that the program will continue to have numerous problems precisely because it lacks a coherent vision for its future... but I don't see the stimulus mentioning that that having or not having serious problems was a benchmark that the government was using to determine competence... does this make sense?

(Unless what I've stated above is supposed to be an "assumption" that we would be allowed to make on the LSAT?)

Let me know if I've made sense.
User avatar
 evelineliu
PowerScore Staff
  • PowerScore Staff
  • Posts: 91
  • Joined: Sep 06, 2021
|
#90210
Hi Leslie,

Answer choice (B) is correct because the report's authors claimed they cannot articulate a clear vision for the program since they needed to deal with their internal problems first. If it were established that the program's problems were caused by the lack of vision, then the author's argument wouldn't hold up.

Hope that makes sense,
Eveline
User avatar
 alexis.la
  • Posts: 13
  • Joined: Jul 14, 2021
|
#90883
Hi there. Reading this thread, I understand why B is correct. But I selected C and I want to explain my thought process to figure out where I went wrong.

My understanding of the stimulus is: The report gets criticized for lacking a coherent vision for the future. The author defends this by saying they need a reputation for competence to get funding, before they take on a 'big' future plan.
Now, the question is asking me to undermine this response. Answer C does this in my interpretation because IF the program once had a coherent vision-- which is what the critic thinks they need, and this vision fell flat then the authors argument of "we need a reputation of competence first" is contradictory. Because they have proven to be incompetent. The stimulus makes it seem like the authors reputation is neutral currently, and needs to become positive, but they're currently at a negative reputation, which is hard to bounce back from.
C undermines the authors response because the excuse "we need to look competent first" BEFORE taking on a 'big' vision isn't applicable if you already took on a big vision in the past. Answer C would force the author to admit "yes, we did once have a coherent vision, and it wasn't sustainable, so that reflects poorly on us" and they would need to alter their response as to why they are lacking A NEW coherent vision, and reputation isn't a good reason.

Get the most out of your LSAT Prep Plus subscription.

Analyze and track your performance with our Testing and Analytics Package.